Moment of Interita about x-axis in Polar Coordinates

Click For Summary
The discussion focuses on calculating the moment of inertia of a plate defined by the curve (x² + y²)² = 9(x² - y²) about the x-axis using polar coordinates. The user successfully transformed the equation into polar form, resulting in r² = 9cos(2θ), and established the moment of inertia formula as an integral involving y². They expressed the integral in polar coordinates but are uncertain about incorporating the bounding curve into their calculations. The user seeks guidance on evaluating the integral to complete the problem. The thread emphasizes the transition from Cartesian to polar coordinates and the challenges of integrating within specified bounds.
jegues
Messages
1,085
Reaction score
3

Homework Statement



A plate with constant mass per unit area \rho is bounded by the curve (x^{2} +y^{2})^{2} = 9(x^{2} - y^{2}) . Find its moment of inertia about the x-axis.


Homework Equations





The Attempt at a Solution



Okay well first I plugged in,

x=rcos\theta,y=rsin\theta

into my equation and simplified a little giving me the following result,

r^{2} = 9cos(2\theta)

Now I know the moment of inertia about the x-axis is defined to be,

\int \int_{R} y^{2} \rho dA

Now if I want to consider this in polar coordinates would it simply be,

\int^{\beta}_{\alpha} \int^{r_{2}}_{r_{1}} r^{2}sin^{2}\theta \rho rdrd\theta

I'm a little confused on how the bounding curve plays into this problem.

Any help?
 
Physics news on Phys.org
Alright I've done some work on the question and basically I've got up to this point in my integration and I get stuck.

This problem is really only an "evaluate the integral" problem at this point.

Does anyone have any suggestions how to compute the rest of this integral? (If it's correct up to that point)
 

Attachments

  • MT101.jpg
    MT101.jpg
    27.6 KB · Views: 501
  • MT102.jpg
    MT102.jpg
    25.8 KB · Views: 470
Question: A clock's minute hand has length 4 and its hour hand has length 3. What is the distance between the tips at the moment when it is increasing most rapidly?(Putnam Exam Question) Answer: Making assumption that both the hands moves at constant angular velocities, the answer is ## \sqrt{7} .## But don't you think this assumption is somewhat doubtful and wrong?

Similar threads

  • · Replies 19 ·
Replies
19
Views
2K
  • · Replies 3 ·
Replies
3
Views
3K
  • · Replies 5 ·
Replies
5
Views
2K
  • · Replies 20 ·
Replies
20
Views
2K
  • · Replies 2 ·
Replies
2
Views
2K
Replies
3
Views
2K
  • · Replies 1 ·
Replies
1
Views
2K
  • · Replies 14 ·
Replies
14
Views
3K
  • · Replies 22 ·
Replies
22
Views
2K
  • · Replies 6 ·
Replies
6
Views
2K